¿Ecuación de Schrödinger de la ecuación de Klein-Gordon?

Uno puede ver QM como un QFT dimensional 1+0, los campos solo dependen del tiempo y, por lo tanto, solo se llaman operadores, y conozco una forma de derivar la ecuación de Schrödinger de la de Klein-Gordon.

Suponiendo un campo Φ con poca energía mi metro con metro la masa de la partícula, definiendo ϕ como Φ ( X , t ) = mi i metro t ϕ ( X , t ) y desarrollando la ecuacion

( 2 + metro 2 ) Φ   =   0

descuidando el t 2 ϕ luego se encuentra la conocida ecuación de Schrödinger:

i t ϕ   =   Δ 2 metro ϕ .

Aún así, no estoy completamente satisfecho con el campo de transición. función de onda, incluso si suponemos que el número de partículas es fijo, y el campo ahora actúa sobre un espacio de Hilbert de dimensión finita (una subparte del primer espacio de Fock completo para un número específico de partículas). ¿Alguien tiene otra proposición/argumento para esta derivación?

Editar: como referencia, los cálculos anteriores se tomaron del libro de Zee, QFT en pocas palabras, primera página en el Capítulo III.5. De manera equivalente, véase Wikipedia .

David Tong tiene una buena derivación/explicación para esto en la sección 2.8 aquí: damtp.cam.ac.uk/user/tong/qft.html Sin embargo, tengo mis propias dudas al respecto, en caso de que tengas las mismas dudas, aquí son hilos que las responden: physicsforums.com/showthread.php?t=709980 physics.stackexchange.com/q/77290
Física relacionada.stackexchange.com/a/734239/226902

Respuestas (2)

Creo que estás mezclando dos cosas diferentes:

  1. Primero, puede ver QM como 0 + 1 (una dimensión temporal) QFT, en la que los operadores de posición (y sus momentos conjugados) en la imagen de Heisenberg desempeñan el papel de los campos (y sus momentos conjugados) en QFT. Puede verificar, por ejemplo, que la simetría rotacional espacial en la teoría mecánica cuántica se traduce en una simetría interna en QFT.

  2. En segundo lugar, puede tomar el "límite no relativista" (por cierto, nombre feo porque la relatividad galileana es tan relativista como la relatividad especial) de la teoría de Klein-Gordon o Dirac para obtener Schrödinger QFT "no relativista", donde ϕ (en su notación) es un campo cuántico en lugar de una función de onda. Hay un capítulo en el libro de Srednicki donde se plantea este tema de una manera sencilla y amena. Allí, también puede leer sobre el teorema de la estadística de espín y la función de onda de los estados de múltiples partículas. Permítanme agregar algunas ecuaciones que espero aclaren eso (estoy usando su notación y, por supuesto, puede haber factores, unidades, etc. incorrectos):

El campo cuántico es:

ϕ d 3 pag a pag mi i ( pag 2 / ( 2 metro ) t pag X )

El hamiltoniano es:

H i d 3 X ( ϕ t ϕ 1 2 metro i ϕ i ϕ ) d 3 pag pag 2 2 metro a pag a pag

La evolución del campo cuántico viene dada por:

i t ϕ [ ϕ , H ] 2 ϕ 2 metro

Los estados de 1 partícula vienen dados por:

| 1 pag d 3 pag F ~ ( t , pag ) a pag | 0

(uno puede definir análogamente estados de múltiples partículas)

Este estado verifica la ecuación de Schrödinger:

H | 1 pag = i t | 1 pag
si y si

i t F ( t , X ) 2 F ( t , X ) 2 metro

dónde F ( t , X ) es la transformada espacial de Fourier de F ~ ( t , pag ) .

F ( t , X ) es una función de onda, mientras que ϕ ( t , X ) es un campo cuántico.

Esta es la teoría libre, se puede agregar interacción de manera similar.

Gracias por la actualización, pero estoy específicamente bajo una operación de límite que me llevaría a un "primer esquema de cuantificación" de un "segundo esquema de cuantificación", alias, ¿es suficiente con el hecho de que recupero una ecuación de Schrödinger y luego puedo construir una corriente de probabilidad conservada con un componente de densidad positivo ( j 0 ) para reinterpretar el campo como una función de onda cuyo módulo cuadrado da probabilidades?
No estoy seguro de lo que estás buscando. F es una función de onda que verifica el Schr. ecuación. El valor esperado de ϕ es también una función que verifica el Schr. ecuación. Entonces, siempre que pueda normalizarlos, obtendrá una interpretación probabilística mecánica cuántica. ¿He respondido a tu pregunta?
@drake: El valor esperado de ϕ no es la forma correcta de extraer la función de onda del campo. La forma correcta es considerar el estado ψ ( X ) ϕ ( X ) dónde ψ es un numero y ϕ ( X ) es el campo no relativista.
@toot: el comentario anterior es la correspondencia entre campos no relativistas y funciones de onda. Si mancha un campo de creación no relativista con una función, produce una partícula con función de onda ψ .
Gracias. Solo estás cambiando mi notación: tu ψ es mi F . El valor esperado es una solución de la ecuación, por lo general se le llama el campo clásico.

En esta respuesta incluimos para mayor claridad los factores correctos de y C en el cálculo del capítulo III.5 de Zee's QFT in a Nutshell.

(4) Φ ( X , t )   =   2 metro Exp ( i metro C 2 t ) ϕ ( X , t ) .

La densidad lagrangiana libre de Klein-Gordon es

(5) L   =   | 1 C t Φ | 2 | Φ | 2 | metro C Φ | 2   = ( 4 )   | ( C 2 metro t i C metro 2 ) ϕ | 2 2 2 metro | ϕ | 2 metro C 2 2 | ϕ | 2   =   | C 2 metro t ϕ | 2 + i 2 ( ϕ t ϕ t ϕ ϕ ) 2 2 metro | ϕ | 2   i 2 ( ϕ t ϕ t ϕ ϕ ) 2 2 metro | ϕ | 2 F o r C     .
La última expresión es la densidad lagrangiana de Schrödinger libre (hasta los términos derivados del espacio-tiempo total), cf. por ejemplo, esta publicación de Phys.SE.